What is the value of the infinite series?

  • Thread starter Thread starter ptolema
  • Start date Start date
  • Tags Tags
    Infinite Sum
Click For Summary
The discussion centers on evaluating the sum of an infinite series defined by a power series centered at zero. Participants suggest using Taylor series and manipulating the series through differentiation to simplify the evaluation process. A specific approach involves rewriting the series in terms of known sums and applying substitutions to find the desired result. The conversation emphasizes the importance of exploring various mathematical techniques to reach the solution. Overall, the thread provides guidance on tackling the evaluation of infinite series through established mathematical methods.
ptolema
Messages
82
Reaction score
0

Homework Statement



evaluate the sum:
sum.jpg


(it may help to think of this as a value of a function defined by a power series.)

Homework Equations



as a function defined by a power series, the function is centered at 0: f(1/3)=
sum.jpg
=1/3+4/9+1/3+16/81+...+n2/3n

The Attempt at a Solution


i'm not entirely clear on how to actually evaluate the value of an infinite series. i could definitely use a few pointers
 
Physics news on Phys.org
hi ptolema! :smile:

(have a sigma: ∑ :wink:)

(not sure about the power series, but … )

have you tried ∑ (n+1)2/3n - ∑ n2/3n ? :wink:
 
ok so how about considering a taylor series about zero as a start
f(a) = \sum_{n=0} f^n(a) x^n

then re-write your series as
0 + 1.\frac{1}{3} + 2^2.\frac{1}{3^2} +3^2.\frac{1}{3^3} +...+ n^2 (\frac{1}{3^n})
 
\sum_{n=1}^{\infty} \frac{x^n}{3^n} = \frac{x}{3-x} for |x| < 3.

Try fiddling around with various manipulations such as differentiating both sides, and try to lead yourself to a series that will drop out your required sum after a suitable substitution for x.
 
Going along the hint by tiny-tim, complete the dots below

\sum_{n=1}^{\infty} \frac{n^2}{3^n} = \sum_{n=0}^{\infty} \frac{(n+1)^2}{3^{n+1}} = \frac{1}{3}\sum_{n=0}^{\infty} \frac{n^2 + 2n +1}{3^n} =...
 
Question: A clock's minute hand has length 4 and its hour hand has length 3. What is the distance between the tips at the moment when it is increasing most rapidly?(Putnam Exam Question) Answer: Making assumption that both the hands moves at constant angular velocities, the answer is ## \sqrt{7} .## But don't you think this assumption is somewhat doubtful and wrong?

Similar threads

  • · Replies 1 ·
Replies
1
Views
2K
  • · Replies 22 ·
Replies
22
Views
4K
  • · Replies 3 ·
Replies
3
Views
2K
  • · Replies 2 ·
Replies
2
Views
1K
Replies
4
Views
2K
  • · Replies 9 ·
Replies
9
Views
2K
  • · Replies 5 ·
Replies
5
Views
3K
Replies
8
Views
2K
  • · Replies 1 ·
Replies
1
Views
1K
  • · Replies 4 ·
Replies
4
Views
2K